0

Hallo eine Frage. 

Ich verzweifle daran den Beweis für die gleichung 

\(2n \le n^2 - 1 \le 2^n - 1\ \text{ für } n \ge 4\) 

zu finden. 

 

Ich habe schon mit \(n=n+1\) versucht \(2(n+1)\) so umzuformen um \(2(n+1)\) ist \( \le (n+1)^2 -1\) zu bekommen und vice versa. 

Aber egal wie ich das mache dreht sich das kleiner bzw. größer gleich zeichen, sodass ich nicht direkt zeigen kann dass \((n+1)^2 -1\) größer ist. 

Diese Frage melden
gefragt

Student, Punkte: 13

 
Kommentar schreiben
1 Antwort
0

Hallo,

die linke Ungleichung ist schnell gezeigt. Klammere es doch mal komplett aus. Du erhälst

\( 2n + 2 \leq n^2 +2n +1 -1 \\ 2n + 2 \leq n^2 +2n \\ 2 \leq n^2 \)

Und dies gilt natürlich für alle \( n \geq 4 \).

Nun musst du noch zeigen \( (n+1)^2 -1 \leq 2^{n+1} - 1 \\ (n+1)^2 \leq 2\cdot 2^n  \)

Als Tipp dafür. Schätze \( (n+1)^2 \) nach oben ab, mit 

\( (n+1)^2 < n^2 + n^2 \)

Wenn du gezeigt hast, das die Abschätzung gilt, gilt auch die Ungleichung, da dann 

\( n^2 + n^2 \leq 2 \cdot 2^n = 2^n + 2^n \\ n^2 \leq 2^n \)

direkt durch die Induktionsvorraussetzung erfüllt wird.

Grüße Christian

Diese Antwort melden
geantwortet

Sonstiger Berufsstatus, Punkte: 29.81K

 

Danke. 


Reichte es die Geltung der Abschätzung so zu zeigen.


\((n+1)^2 = n^2 + 2n+1-1 = n^2 + 2n < n^2 + n^2, \text{ da n^2 > 2n}\)

  ─   berkalp 18.02.2019 um 14:08

Genau, aber das gezeigte gilt nur für \( n \geq 2 \). Da wir aber es aber nur für \( n \geq 4 \) zeigen müssen, gilt die Abschätzung. Würde es einmal kurz erwähnen :)


Grüße Christian

  ─   christian_strack 19.02.2019 um 18:35

Kommentar schreiben